Investments

अब Quizwiz के साथ अपने होमवर्क और परीक्षाओं को एस करें!

Technical Analysis

- Charting - Dow theory - Breadth of the market - Advance Decline Line

Hindsight bias leads to:

- Complicated clients wanting to know why their advisors did not anticipate events that the client "knew" would happen

Eurodollars

- Deposits in foreign banks that are denominated in US dollars

Bankers Acceptance

- Facilitates imports/exports - Maturities of 9 months or less - Can be held until maturity or traded

Value-weighted index

- Incorporates market capitalization - S&P 500 - Russel 2000 -Wilshire 5000 -EAFE

Treasury Bills

- Maturities of 30, 91, and 182 days - 4, 13, 26, and 52 weeks - Denomination of $100

Commercial Paper

- Short term loans between corporations - Maturities of 270 days or less -Does not have to register with the SEC - Denominations of $100,000 and are sold at a discount

Preferred Stock

-Dividend does not flucuate - Price is more closely tied to interest rates -Tax advantage: corporations receive the highest deductions 70,80, or 100% based on percentage of ownership of the company

Unit Investment Trust

-issues "Units" not shares - Passively managed - Self-liquidating

1 standard deviation equals?

68%

2 standard deviations equal?

95%

3 standard deviations equals

99%

Types of Unsystematic Risk?

ABCDEFG -Accounting risk -Business risk -Country risk -Default risk -Executive risk -Financial risk -Government risk

Intrinsic Value of an Option contract -Call Option -Put option -Time value

Call option: The stock price - Strike price Put option: Strike price - The Stock price Time value: Premium - intrinsic value Intrinsic value cannot be less than 0

coefficient of variation

Coefficient of variation is useful in determining which investment is riskier when investments have different average returns - The higher the CV the more risky an investment per unit of return - CV=standard deviation/ average return

Fundamental Analysis

Considers -Financial statement analysis - Economic data - Ratio analysis

Price-weighted Index

Does not incorporate market capitalization - DJIA

Leptokurtic (leptokurtosis)

High peak and fat tails - Lepto distribution reflects the tendency of observations to fall closely around the mean creating a peaked distribution at the mean with thicker tails. - If historical returns indicate leptokurtosis then there is much more reserved variation in periodic returns but higher probability of large mutli-sigma deviations (fat tails)

Weak Form

Historical information will not help investors - rejects technical analysis - Fundamental analysis and inside information will help an investor achieve above-average returns

Dividend Discount Model tips -Required rate of return and stock price is INVERSELY RELATED -expected Dividend and stock price is SAME DIRECTION

If the required rate of return decreases, the stock price will increase If the dividend is expected to increase, the stock price will increase If the required rate of return increases, the stock price will decrease If the dividend is expected to decrease, the stock price will decrease

Overconfidence leads to:

Increased risk taking - Overtrading

Covariance

Is the measure of two securities combined and their interactive risk. - is a measure of relative risk

Platykurtic

Low peak, and thin tails

Margin Call Equation

Margin call = Loan / 1 minus maintenance margin

P/E Ratio - P/E Ratios are a useful tool used to value stock if the firm pays no dividends

P/E = Price per share/EPS or Price of share = P/E X EPS

Types of Systematic Risks?

PRIME -P purchasing power risk -R reinvestment risk -I interest rate risk -M market risk -E exchange rate risk

Morningstar

Ranks mutual funds using 1 to 5 stars - 1 star represents the lowest ranking; - 5 stars represents the highest ranking

Value line

Ranks stocks on a scale of 1 to 5 for timeliness and safety - ranking of 1 is the highest rating ( Signal to buy) -Ranking 5 is the lowest ranking (Signal to sell) -

Jasmine has a large paper profit in her Amalgamated Corporation shares, currently at $46 per share. She is happy with the stock, but realizes that a good thing CANNOT go on forever. She bought the stock so inexpensively that she is not worried about the downside. If she is willing to sell at $50, what strategy could you recommend to her? A) Buy $50 call options. B) Sell $50 call options. C) Buy $50 put options. D) Sell $50 put options.

Rationale The answer is "B." She gains the premium from selling the call, and if the price rises, at or above the strike price of $50, her stock will be called away at $50. "C" would be a good choice, but she is not worried about the downside risk.

Which of the following statement(s) regarding bond swaps is/are true? I. A substitution swap is designed to take advantage of anticipated and potential yield differentials between bonds that are similar with regard to coupons, rating, maturities, and industry. II. Rate anticipation swaps utilize forecasts of general interest rate changes. III. The yield pickup swap is designed to alter the cash flow of the portfolio by exchanging similar bonds having different coupon rates. IV. The tax swap is made to substitute current yield in place of capital gains. A) I, II and III only. B) I and III only. C) II and IV only. D) IV only.

Rationale The correct answer is "A." All statements are correct except for IV. The tax swap replaces bonds with offsetting capital gains and losses.

During the peak of the economic cycle, which of the following should one undertake? I. Sell debt instruments II. Begin allocations to cash positions III. Buy debt instruments IV. Sell gold and real assets A) I and II only. B) II and III only. C) III and IV only. D) I and IV only.

Rationale The correct answer is "A." At the peak of the cycle, as the economy has reached full steam, it is an excellent time to sell not buy fixed (and generally lower return) instruments. It is also an excellent time to begin appropriations to cash in preparation for opportunities that may arise. Since inflation is still on (and rising after the peak), it often proves to be a good time to acquire metals rather than sell them.

John Gray would like to know the expected return on his portfolio. He owns: 5 options on ABC, Inc. with a total current market value of $2,300 10 shares of NOP with a total current market value of $4,725 7 YAK bonds with a total current market value of $6,850 John expects a total return of 28.0%, 8.0%, and 13.0% respectively on the investments. What is the overall weighted average expected return on John's portfolio? A) 13.8% B) 14.14% C) 15.6% D) 16.3%

Rationale The correct answer is "A." Based on the calculation of the weighted return of each of the holdings. ABC $2,300 NOP $4,725 YAK $6,850 Total $13,875 ABC (2,300 ÷ 13,875) x .28 = .0464 NOP (4,725 ÷ 13,875) x .08 = .0272 YAK (6,850 ÷ 13,875) x .13 = .0642 Total .1378

Which of the following represent the minimum information needed to calculate the weighted average rate of return for a portfolio? I. Current market price of each security. II. Price paid for each security. III. Number of shares of each security. IV. Total portfolio value. V. Percent return of each security. A) I, III and V only. B) II, III and IV only. C) III, IV and V only. D) I, IV and V only.

Rationale The correct answer is "A." Current market price times number of shares gives the total value of each investment. These can be summed to calculate the total portfolio value. Then, the total value of each investment is divided by the total portfolio value and multiplied by the percent return of each security to calculate the weighted return. These weighted returns added together give the weighted average portfolio rate of return.

Which one of the following factors would be the strongest indication that interest rates might rise? A) Selling of dollar-denominated assets by foreign investors. B) Decreasing United States government deficits. C) Decreasing rates of inflation. D) Weak credit demand by the private sector of the United States economy.

Rationale The correct answer is "A." Foreigners selling dollar-denominated assets are preparing to take advantage of higher rates by increasing their liquidity. The rest signal a decrease in rates.

Bob Conrad's investment portfolio consists of several types of stocks, bonds, and money market instruments. The portfolio has an overall standard deviation of 12%, a beta of 1.06, and a total return for the year of 11%. Bob is considering adding one of two alternative investments to his portfolio. Stock A has a standard deviation of 13%, a beta of .87, and a correlation coefficient with the portfolio of .6. Stock B has a standard deviation of 11%, a beta of .97, and a correlation coefficient of .95. Which stock should Bob consider adding to his portfolio, and why? A) Stock A, because it has a lower correlation coefficient. B) Stock A, because it has a lower beta than that of the portfolio. C) Stock B, because it has a lower standard deviation than that of the portfolio. D) Stock B, because it has a higher correlation coefficient.

Rationale The correct answer is "A." In the process of adding new investments to a portfolio, the lowest correlation coefficient makes the best addition. Closest to negative one (-1) is always best.

Which of the following elements of risk in mortgage-backed securities can be difficult to determine? I. Actual maturity is not known with certainty. II. Mortgage rates vary between the different investment pools. III. Actual cash flows are not known with certainty. IV. Government guarantees make the determination of an appropriate discount rate for calculating their present value difficult. A) I and III only. B) I and IV only. C) II and III only. D) II and IV only.

Rationale The correct answer is "A." Lack of a definite maturity date and uncertain cash flows are the elements of risk in mortgage-backed securities.

What is one implication of the efficient market hypothesis? A) Consistently superior performance is rare. B) The weak form suggests that security prices do NOT adjust to new information. C) The random walk hypothesis is invalidated. D) Anomalies are perceptions but do NOT actually exist.

Rationale The correct answer is "A." One cannot consistently earn abnormal returns. Over time, these superior returns will be reversed and overall gain is consistent with the general market.

What is the weighted average beta of the following portfolio? Stock L has a beta of 1.45 and has a value of $60,000, 10% of the portfolio, stock M has a value of $125,000 with a beta of .93, while stock N has a value of $240,000, 40% of the portfolio with a beta of .65, and stock O, with a 2.2 beta has a dollar value of $175,000. A) 1.24 B) 1.31 C) 1.54 D) 1.76

Rationale The correct answer is "A." Please be certain to avoid rounding errors in arriving at the correct solution. For weighted average beta, use only the two places to the right of the decimal. Beta Value Weighting Weighted Beta L 1.45 60,000 60,000 ÷ 600,000 = 10% 1.45 X .10 = .145 M 0.93 125,000 125,000 ÷ 600,000 = 20.83% .93 X .2083 = .1938 N 0.65 240,000 240,000 ÷ 600,000 = 40% .65 X .40 = .26 O 2.2 175,000 175,000 ÷ 600,000 = 29.17% 2.2 X .2917 = .6417 600,000 1.24

If one of your clients has a profitable long position in an oranges futures contract and does nothing as the contract expires, what should she expect to occur? A) The oranges will be delivered to her. B) She will receive a substantial check as soon as the account is settled. C) Her contract will expire worthless unless she takes some action. D) Her broker will arrange for sale of the oranges in an appropriate market.

Rationale The correct answer is "A." Positions in futures contracts are closed by taking an equal and opposite position. One who is long on a contract at the expiration should expect delivery of the commodities at the stated contract price. It is the buyers responsibility, but in this case, we could say the broker was remiss in his or her duties!

An investor who rebalances her portfolio frequently to take advantage of perceived opportunities in other market sectors is using which one of the following types of asset allocation? A) Tactical. B) Strategic. C) Passive. D) Hybrid.

Rationale The correct answer is "A." Strategic asset allocation is concerned with allocating the wealth of a client among various asset classes, consistent with the clients' investment objectives, time horizons and risk preferences. Tactical asset allocation is concerned with shifting wealth between asset classes to take advantage of expected price level changes (timing) arising from broad movements in the business cycle.

Walt Drizzly stock is currently trading at $45 and pays a dividend of $3.50. Analysts project a dividend growth rate of 5%. Your client, Toby Benjamin, requires a rate of 12% to meet his stated goal. Toby wants to know if he should purchase stock in Walt Drizzly. A) Yes, the stock is undervalued. B) No, the stock is overvalued. C) No, the required rate of return is higher than the projected growth rate. D) Yes, the required rate is higher than the expected rate.

Rationale The correct answer is "A." The intrinsic value is: V = (D1 / r - g), therefore V = (3.50 x 1.05) / (.12 - .05), V = $52.50 compared to the selling price of $45. Therefore the stock is undervalued.

A put option with a strike price of $50 is selling for 3 1/2 when the market price of the underlying stock is $52. The intrinsic value of the put is: A) 0 B) 1 1/2 C) 2 D) -2

Rationale The correct answer is "A." The intrinsic value of a put option is strike price minus stock price (50-52). If the answer arrived at is negative, then the intrinsic value is zero, and the value of the put is entirely related to the time component. In the case of a call, the intrinsic value is the stock price minus the strike price.

The primary reason for using a ladder bond strategy is to: A) Lower overall interest rate risk. B) Achieve greater capital gains as the yield curve changes shape. C) Avoid the "wash sale" rule. D) Immunize the bond portfolio.

Rationale The correct answer is "A." The ladder bond strategy staggers maturities and in doing so, reduces the exposure to interest rate risk. "B" is incorrect because longer term maturities experience the biggest percentage increase when interest rates decrease. Laddering bonds requires purchasing short and intermediate term bonds, along with long term bonds. "C" is incorrect because laddering bonds to avoid the wash sale rule is not the primary objective, reducing interest rate risk is the primary objective. "D" is incorrect because bond immunization suggests the portfolio has eliminated interest rate and reinvestment rate risk. Laddering bonds does not eliminate either interest rate or reinvestment rate risk, however it does reduce interest rate risk.

John Risotto has a cash need at the end of nine years. Which of the following investments best meets this need and serves to immunize the portfolio initially? I. An 11-year maturity coupon bond. II. A 9-year maturity coupon Treasury note. III. A series of Treasury bills. A) I only. B) II and III only. C) II only. D) I and II only.

Rationale The correct answer is "A." The process of portfolio immunization entails not maturity of a security, but its duration. Duration is based on coupon rate. The larger the coupon payment, the shorter the duration. This being the case, a bond generally pays higher interest than a note, and a note pays higher than short-term Treasury bills. Given this information, one could reasonably expect a shorter duration (than time to maturity), while receiving better immunization from the bond.

Which of the following is NOT a premium factor that would be considered part of the nominal rate of interest? A) Economic premium. B) Default premium. C) Liquidity premium. D) Risk free rate of interest.

Rationale The correct answer is "A." There is no such thing as an economic premium. All of the other premiums added to the risk free rate equal the nominal (or stated) rate.

You are faced with several fixed income investment options. Which of these bonds has the greatest reinvestment rate risk? A) A U.S. Treasury bond with an 11.625% coupon, due in five years with a price of $1,225.39 and a yield to maturity of 6.3%. B) A U. S. Treasury strip bond (zero-coupon) due in five years with a price of $735.12 and a yield to maturity of 6.25%. C) A corporate B-rated bond with a 9.75% coupon, due in five years with a price of $1,038.18 and a yield to maturity of 8.79%. D) A corporate zero coupon bond due in 5 years with a price of $750 and a yield to maturity of 5.9%.

Rationale The correct answer is "A." This is due to the high coupon and lack of similar rates currently.

John Henry has requested information regarding the risk involved in his overseas investments. He has given you the following information: Monrovia Investment: Standard deviation of 17.5%, Rate of return = 16.3% and Weight in portfolio of 60% Zimbabwe Investment: Standard deviation of 24.9%, Rate of return = 32.1% and Weight in portfolio of 40% Harry wants to know what the portfolio risk (as measured by portfolio deviation) is if the correlation coefficient of these two investments is negative .35 (-.35). A) 11.6% B) 9.7% C) 5.8% D) 1.36%

Rationale The correct answer is "A." Use the formula for portfolio deviation to calculate the correct answer to this problem. S p = √ (.60)2(.175)2 + (.40)2(.249)2 + 2(.60)(.40)(.175)(.249)(-.35) S p = √ .0110 + .0099 - .0073 S p = √ .0136 S p = .1165

Bond A has a 6% annual coupon and is due in 2 years. Its value in today's market is $900. Bond B has a 10% annual coupon and is due in 4 years. It is priced to yield 12%. Bond C is a zero-coupon bond priced to yield 11% in 8 years. Assuming the duration of Bond A is 1.94 years, which of the following statements about the effect of a 1% decline in interest rates is true? A) Bond C, having a longer duration that Bond A, would have a larger percent increase in price than Bond A. B) The percent change in price of a bond is independent of the duration of a bond. C) It is NOT possible to determine the percent change in price of Bond A versus Bond C because the duration of Bond C is NOT given. D) Bond A would have a greater percent change in price than Bond C because it has a shorter duration.

Rationale The correct answer is "A." Use the rule of thumb and the correct answer becomes clear as a result. Multiply the change of rate by the duration to arrive at the percent change in price (see formula sheet for percent change formula). For each percent of change, multiply by the duration to arrive at the total percent change. Then calculate that percentage into a price change.

An analysis of the monthly returns for the past year of a mutual fund portfolio consisting of two funds revealed the following statistics: Fund A total return = 18% Fund A Standard deviation = 23% Fund A Percentage of portfolio = 35% Fund B total return = 11% Fund B Standard deviation = 16% Fund B Percentage of portfolio = 65% The Correlation Coefficient ("R") between the two funds equals .25. What is the standard deviation of the portfolio? A) 13.16% B) 14.66% C) 18.45% D) 19.50%

Rationale The correct answer is "B." s p = √(.35)2(.23)2 + (.65)2(.16)2 + 2(.35)(.65)(.23)(.16)(.25) s p = √.0065 + .0108 + .0042 s p = √.0215 s p = .1466

In the mutual fund industry, 12b-1 fees are charged as part of: I. Fund management fees. II. Distribution fees. III. Commissions for sales. IV. Legal fees and expenses. A) I only. B) II only. C) I and III only. D) I, II and IV only.

Rationale The correct answer is "B." 12b1 fees are used for marketing and distribution costs. All other costs, such as legal, accounting and analysis are paid through management fees. Commissions are paid using either a front load or a back load.

Developing cash flow projections and valuations for real estate can be difficult due to: A) A lack of comparable figures for other properties in the area. B) Changes in demographic and economic variables. C) Different financing methods amongst prospective purchasers. D) A lack of standardized methods for objectively evaluating an investment in a market that is considered inefficient

Rationale The correct answer is "B." Cash flow projections and comparable equity capitalization rates are easily obtained for a valid comparison. The difficulty is one of the unpredictability of changes in economics and demographics which directly impact values. Real estate valuation models such as one using net operating income, adjust for variations in real estate financing.

Your client holds a diversified equity portfolio, and has asked for your opinion on what is the most important factor to consider as he prepares to add additional equity securities. You tell him: A) Standard deviation of the new additions. B) Correlation of the new securities to the portfolio. C) Coefficient of variation of the new securities. D) Beta of the new securities to the existing portfolio.

Rationale The correct answer is "B." Correlation of securities is always the strongest determinant as to what should be added to a portfolio.

Jennifer has asked you if you would advise her regarding several different types of investments. Her preference would be for an investment where she can simply put a fixed number of dollars into an investment and not worry about it. She wants the following: completely tax-advantaged investments, a moderately competitive interest rate, with relative safety, and very low fees. She would eventually like to be assured of getting her principle back, but does not require a great deal of liquidity. Which of the following would you recommend to her in order to best meet her goals? A) Municipal Bond Mutual Fund. B) Municipal Bond Unit Trust. C) High Yield Money Market Fund. D) Tax Free Money Market.

Rationale The correct answer is "B." Due the tendency of Muni-Bond Fund managers to attempt to maximize profits by buying and selling various bonds, there are generally taxable gains to be dealt with in most of these funds. High Yield and Government Bond funds offer no tax advantage. Tax Free Money fund is highly liquid and the client doesn't require much liquidity.

A yield curve normally is upward sloping because: A) Long-term rates must be higher to compensate for higher expected future tax rates. B) Long-term bonds are, by their nature, more risky than short-term bonds. C) Higher long-term rates reflect inflationary expectations. D) Short-term bonds have a lower level of event risk.

Rationale The correct answer is "B." Short-term rates are lower because of the lower risk associated with them. The longer an investment ties up an investor's capital, the higher the rate must be to offset this risk. Inflation expectations may be lower in the future, resulting in a downward (inverted) sloping yield curve.

Margin accounts involve security transactions performed using some amount of capital borrowed from the brokerage firm as well as some of the investor's own capital. The entity that establishes the initial margin requirement is the: A) Securities and Exchange Commission. B) Federal Reserve. C) National Association of Securities Dealers. D) Brokerage firm with which an investor is dealing.

Rationale The correct answer is "B." The Federal Reserve sets margin requirements for all security transactions.

A stock that has produced superior earnings and rates of return but has gone mostly unnoticed by securities analysts and is often considered underpriced is said to benefit from the: A) P/E effect. B) Neglected firm effect. C) Small firm effect. D) Low price/sales effect.

Rationale The correct answer is "B." The neglected firm effect is one of the market anomalies. This anomaly is said to exist because the security in question is allowed greater potential for movement as a result of the lack of scrutiny by analysts.

When evaluating the return of two investment managers, the performance measurement approach generally used is the: A) Internal rate of return. B) Time-weighted return. C) Value-weighted return. D) Dollar-weighted return.

Rationale The correct answer is "B." Time-weighted rate of return is a compounded rate of return calculation that allows for a more accurate comparison based on the security's cash flows. Internal rate of return is a compunded rate of return, but we need to be more specific whether to consider the security's cash flows or the investor's cash flow.

Which of the following is/are characteristics of a municipal bond unit investment trust? I. Additional securities are NOT added to the trust. II. Shares may be sold at a premium or discount to net asset value. III. Shares are normally traded on the open market (exchanges.) IV. The portfolio is self-liquidating. A) I only. B) I and IV only. C) II and III only. D) II and IV only.

Rationale The correct answer is "B." Unit investments do not make additions to investments once the trust has been structured. Shares are not bought or sold after structuring and the portfolio is self-liquidating.

You are faced with several fixed income investment options. Which of these bonds has the greatest interest rate risk? A) A U.S. Treasury bond with an 11.625% coupon, due in five years with a price of $1,225.39 and a yield to maturity of 6.3%. B) A U.S. Treasury strip bond (zero-coupon) due in five years with a price of $735.12 and a yield to maturity of 6.25%. C) A corporate B-rated bond with a 9.75% coupon, due in five years with a price of $1,038.18 and a yield to maturity of 8.79%. D) A U.S. T-bill selling for $950 due in six months.

Rationale The correct answer is "B." With the term being equal, the bond with the lowest coupon will have the biggest duration. The bigger the duration, the price more sensitive the bond is to interest rate changes. Bond B has the lowest coupon, zero.

The Performance Fund had returns of 19% over the evaluation period and the benchmark portfolio yielded a return of 17% over the same period. Over the evaluation period, the standard deviation of returns from the Fund was 23% and the standard deviation of returns from the benchmark portfolio was 21%. Assuming a risk-free rate of return of 8%, which one of the following is the calculation of the Sharpe index of performance for the Fund over the evaluation period? A) .3913 B) .4286 C) .4783 D) .5238

Rationale The correct answer is "C." (.19 - .08) / .23 = .4783

Which of the following best describes a long hedge position? A) The investor is short the underlying commodity and short the futures contract. B) The investor is long the underlying commodity and long the futures contract. C) The investor is short the underlying commodity and long the futures contract. D) The investor is long the underlying commodity and short the futures contract.

Rationale The correct answer is "C." A long position in a futures contract is when the investor buys a futures contract. A short position in a futures contract is when the investor sells a futures contract. A long hedge means that the investor owns (buys) the futures contract to insure a certain price of a commodity that he or she does not yet own. Hedging is taking an opposite futures position than the investor's inherent underlying position.

Which index should Jan use as a benchmark when evaluating the performance of her XYZ mutual fund? Index #1 Beta = .75 r-squared = .80 Index #2 Beta = 1.1 r-squared = .90 Index #3 Beta = 1.25 r-squared = .95 Index #4 Beta = 1.5 r-squared = .50 A) Index #1. B) Index #2. C) Index #3. D) Index #4.

Rationale The correct answer is "C." Always select the index that explains "the most" of a funds return. 95% of the return, as measured by r-squared, for fund XYZ is explained by Index #3.

Match the investment characteristic(s) listed below which describe(s) a unit investment trust. A) Passive management of the portfolios. B) Self-liquidating investments usually holding bonds. C) Both "A" and "B." D) Neither "A" nor "B."

Rationale The correct answer is "C." Both statements are correct because a UIT typically holds municipal bonds until maturity. UITs can also own equities.

The following investment return will result in what dollar weighted return? An initial outlay of $50,000, with three years of additional outflows of $10,000 each, and inflows as follows: $0 the first year, $20,000 in years 2 and 3, and sale of the property at the end of year 3 for $75,000. A) 27.64% B) 14.04% C) 18.32% D) 20.67%

Rationale The correct answer is "C." CF0 = <50,000> CF1 = 0 - 10,000 = <10,000> CF2 = 20,000 - 10,000 = 10,000 CF3 = 20,000 - 10,000 + 75,000 = 85,000 IRR = ?

The Chesapeake Bay apartment complex contains 60 one-bedroom apartments renting for $650 per month. In addition, the complex generates $625 per month from laundry, parking, and vending machines. Vacancy and collection losses have averaged 8% of Potential Gross Income (PGI) and are expected to continue at about the same rate in the future. Annual expenses totaling $117,000 include: Property taxes = $22,000 Property management = $15,000 Maintenance and utilities = $36,000 Swimming pool = $13,000 Professional fees = $8,000 Other expenses = $23,000 What is the property's net operating income? A) -$74,220 B) $175,628 C) $320,460 D) $348,500

Rationale The correct answer is "C." Gross rental receipts ($650 x 60 x 12) = $468,000 plus non-rental income ($625 x 12) = $7,500 equals potential gross income (PGI) ($468,000 + $7,500) = $475,500. PGI minus vacancy and collection losses ($475,500 - (.08 x $475,500) ) = $437,460 equals Effective Gross Income (EGI). EGI minus expenses equals net income ($437,460 - $117,000) = $320,460. Net Income + Interest + Depreciation = New Operating Income 320,460 + 0 + 0 = 320,460 Since there is no information regarding interest or depreciation expense, Net income = NOI in this question.

Your client, Bill Brown, is an investor in ONLY dividend paying stocks. He buys them in time to catch the dividend and then sells them. You have a stock that you have researched, Gamma Globe, and it generally pays very high dividends relative to its price. What will you advise Bill to do in regard to this stock? A) Wait until the stock goes ex-dividend to make the purchase, and sell immediately thereafter. B) Get in on Gamma Globe's action for the long haul, that is "buy and hold." C) Purchase the stock by holder-of-record date, and sell when the price has rebounded. D) It doesn't matter when Bill buys it, but once purchased, Gamma Globe should be held only in preparation for almost certain subsequent price increases.

Rationale The correct answer is "C." It is the only answer that matches the client's express strategy. Buy and hold is not what the client wants, and ex-dividend dates are too late to get in on the dividend action which the client seeks.

The Securities Act of 1933: A) Requires the registration and provides for regulation of investment advisors. B) The1934 Act regulates securities in the secondary markets. C) Regulates both initial public offerings and subsequent secondary offerings by a public company. D) Established the organized securities exchanges.

Rationale The correct answer is "C." Option "A" - The Investment Advisors Act of 1940 regulates advisors. Option "B" is incorrect because the Act of 1934 regulates the secondary market. Option "C" - The Act of 1933 regulates both IPOs and secondary offerings. Option "D" - The organized exchanges and previously issued securities are governed by the Securities and Exchange Act of 1934.

In order to determine whether a stock is overvalued or undervalued, a planner would use which of the following formulas? A) Security market line formula. B) Expected rate of return formula. C) Intrinsic value formula. D) Required rate of return formula.

Rationale The correct answer is "C." Options "A", "B", "C" and "D" are real formulas. Options "A" and "D" are the CAPM formula, and "B" is used to calculate what one should expect as an interest rate of return, not a price.

Herkimer Green has just inquired about purchasing callable bonds. You explain what this means, and you also explain the negative aspects for investors with regard to callable issues. These include: I. The uncertainty about the amount of payments to be made to the bondholders. II. The price risk for the investor. III. The inflation risk to the bondholder. IV. The reinvestment risk faced by the bond investor. V. The investor's liquidity risk. A) I only. B) II and III only. C) I and IV only. D) II, IV and V only.

Rationale The correct answer is "C." Price risk is not a callable bond concern; in fact, often when called a premium is paid. Inflation risk, where the purchasing power of the bond is affected might seem like a good choice, but because the bond is being held to call in this case, it is not a concern and liquidity risk is eliminated. When the bond is called, an investor does not need to worry about selling the bond.

My margin requirements are 50% initial margin and 25% maintenance margin. I purchase a total of 200 shares at $100 per share using full margin amount for the 200 share purchase. Shortly thereafter, share prices fall to $50 per share. What will my margin call be? A) $1,000 B) $1,500 C) $2,500 D) $5,000

Rationale The correct answer is "C." Required: $50 x .25 = 12.50 Actual: 50 - 50 = 0 $12.50 per share x 200 = $2,500

As a measure for risk, the Capital Market Line (CML) uses the: A) Risk free rate of return. B) Beta of the market. C) Standard deviation of the market. D) Portfolio weighted beta.

Rationale The correct answer is "C." The CML (Capital Market Line) uses standard deviation, while the SML (Security Market Line) uses the beta as its "risk" measurement.

Of the following indexes, which is the only one that uses the geometric average to compute its daily value? A) NASDAQ Index. B) Wilshire 5000 Index. C) Value Line Average. D) Dow Jones Industrial Average.

Rationale The correct answer is "C." The NASDAQ, the NYSE Composite, and the Wilshire all use value weighted average, while the Dow Jones Industrial is a simple price weighted average. Only Value Line uses the geometric average.

A convertible bond your client is interested in buying has a 8% coupon rate, a conversion ratio for 100 shares, while market rates on similar issues are currently 12%. It pays interest twice a year, has a par value of $1,000 with a maturity of 20 years. The current stock price is $8.50 per share. What price should your client expect to pay? A) $1,125 B) $1,000 C) $850 D) $699

Rationale The correct answer is "C." The calculated value as a debt instrument here is $699.07, but when the conversion value puts the rate at $850, you can be certain that the bond is selling in the market place at its convertible value of $850, not its debt value of $699. CV = (1,000 ÷ CP) x Ps $850 OR N = 20 x 2 = 40 i = 12 ÷ 2 = 6 PV = ? PMT = (.08 x 1,000) ÷ 2 = 40 FV = 1,000 PV = 699.07 The conversion price is not given; however, (1,000 ÷ CP) is the conversion ratio, which is given (100 shares). Therefore: CV = 100 shares x Ps CV = 100 x 8.50 CV = $850

The value of the convertible bond as a debt instrument does not depend on which of the following? A) The bond's coupon. B) Current interest rates. C) The conversion price of the bond. D) The term of the bond.

Rationale The correct answer is "C." The value of the bond as a debt instrument is considered separately from its convertibility and is calculated using the bond formula or the present value methodology on the calculator. However, should the conversion value be greater than the "debt value" of the bond, it will sell for the higher price.

A yield curve can be described as a curve that: A) Depicts the current yield on government debt. B) Slopes upward as the years to maturity increase. C) Shows the term structure of interest rates on government debt. D) Offers greater potential yield as the maturity approaches the present.

Rationale The correct answer is "C." The yield curve demonstrates graphically the relationship between long-term and short-term government debt.

A client has bought a stock for $40 per share. At the end of the first year, she purchases another share at $43 per share. At the end of the second year with the share price of $48, she sells her shares. Along the way, at the end of each year, she received a $2 per share dividend. What is the time-weighted return on her investment? A) 9.53% B) 13.5% C) 14.3% D) 16.6%

Rationale The correct answer is "C." This is simply an uneven cash flow problem. CF0 = <$40> CF1 = $2 CF2 = $50 IRR = 14.33% Note: Since this is a time weighted return, we are only concerned about the security's cash flow. Therefore, we ignore the second purchase at $43 per share.

What is the return that your client should expect from a security that last year returned 11.7% with a standard deviation of .146, a beta of 1.2, when the overall market return has been 10.93%, and U.S. Treasury issues are currently delivering around 3.56%? A) 14.6% B) 13.3% C) 12.4% 12.4% D) 11.7%

Rationale The correct answer is "C." Using the CAPM one can calculate this answer. Standard deviation and last years return are merely distractors. ER = Rf + B (Rm - Rf) ER = .0356 + 1.2 (.1093 - .0356) ER = .0356 + .0884 ER = .1240 =

Ben Robinson owns two stocks in equal dollar amounts with the following characteristics: Stock Expected Returns Standard Deviation Beta ABC 9.5% 13% 0.93 XYZ 12.0% 18% 1.12 The correlation of returns between the two stocks is -.89. Which of the following statements are true? I. The standard deviation of the portfolio is the average of the standard deviations of the two stocks. II. The standard deviation of the portfolio is less than the standard deviation of either of the two stocks. III. A negative correlation achieves a better portfolio diversification than would a positive correlation. IV. The diversification achieved by a combination of ABC and XYZ in a portfolio depends more on the average portfolio beta than on the correlation. A) I and III only. B) I and IV only. C) II and III only. D) II and IV only.

Rationale The correct answer is "C." We are combining two stocks into a portfolio. Therefore we know the return will be the weighted average return of each of the stocks and the standard deviation of the portfolio will be less than the weighted average return of each of the stocks since the correlation is less than 1. Statement I is incorrect per the statement above which eliminates answers A and B. We have already narrowed the problem down to 50/50. Statement II has to be correct since only C and D are possible answers. This is proven in the comment above. Statement III is a correct statement since a correlation less than 1 means the stocks actually move opposite each other to some degree. This provides our portfolio with enhanced diversification over having two stocks with positive correlation. Statement IV is incorrect since the covariance / correlation is the most important factor regarding diversification. So even if you didn't know III was true you could narrow it down by eliminating I and IV as choices.

A rise in the price of the Japanese Yen in relation to the U.S. Dollar results in: A) A devaluation of the Yen. B) Excess reserves in the U.S. current account. C) A revaluation of the Yen. D) A negative balance of payments.

Rationale The correct answer is "C." Were the Yen to fall in value against the dollar, this would constitute a devaluation, but when it costs more dollars to buy a Yen, this is considered an appreciation or revaluation of the Yen.

If the market risk premium were to increase, the value of common stock (everything else being equal) would: A) NOT change because this does NOT affect stock values. B) Increase in order to compensate the investor for increased risk. C) Increase due to higher risk-free rates. D) Decrease in order to compensate the investor for increased risk.

Rationale The correct answer is "D." A need for higher return to meet the onset of higher risk would drive the price of a security down (all other things being equal).

Eddie Bauer bought a tax-exempt Original Issue Discount (OID) bond in November of 1998. Which of the following statements is/are true? I. The bond basis increases at a set rate each year. II. The difference between maturity value and the original issue discount price is known as the OID. III. The bond's earnings are treated as exempt interest income. IV. The bond was issued at a discount to its par value. A) II and III only. B) I and IV only. C) I, II and IV only. D) I, II, III and IV.

Rationale The correct answer is "D." All of the above statements are descriptions of the Original Issue Discount bond.

Whenever there is a cash dividend issued on an underlying stock, the price (or premium) for a call option available on that stock tend to be: A) Unaffected. B) Higher. C) Volatile. D) Lower.

Rationale The correct answer is "D." Cash dividends will generally tend to drive the price of the underlying security lower and along with it, the call option prices.

Company A has 60% debt and 40% equity; Company B has 20% debt and 80% equity. Assume both companies have the same dollar amount of assets and net income before interest and taxes. Which one of the following statements is true? A) The unsystematic risk for the two companies is about equal. B) Company A's tax obligation will exceed Company B's. C) The company with the higher return on equity should be purchased by a risk-averse investor. D) The return on equity for Company A can be expected to exceed the return on equity for Company B.

Rationale The correct answer is "D." Company A has a smaller amount of its assets financed by equity, therefore, with the same earnings in net income as Company B, the level of return on the equity of Company A would be greater. For example: A: Assets $10M; Liabilities $6M; Equity $4M; EBIDTA = $1M; I/Y = 5% NI = $700K; ROE = $700,000 ÷ $4M = 17.5% B: Assets $10M; Liabilities $2M; Equity $8M; EBIDTA = $1M; I/Y = 5% NI = $900K; ROE = $900,000 ÷ $8M = 11.25%

Which of the following are characteristics of Government National Mortgage Association (GNMA) securities? I. Investors are guaranteed, by the U.S. government, against losses arising from investments in GMNA securities. II. The amount received by the investor each month may vary due to prepayment by homeowners. III. The realized yield on the certificates can be somewhat variable because of the principal prepayments. IV. If mortgage rates decrease, prepayments may increase. A) I and IV only. B) II and IV only. C) I, II, and III only. D) II, III, and IV only.

Rationale The correct answer is "D." GNMA is "on budget" agency debt. This means the pools of mortgages are backed by the full faith, credit, and taxing power of the U.S. government itself. The government backs the issue against default, NOT against investor loss through poor timing or poor choices. It should also be noted no U.S. government agency debt has ever defaulted.

The Chesapeake Bay apartment complex contains 60 one-bedroom apartments renting for $650 per month. In addition, the complex generates $625 per month from laundry, parking, and vending machines. Vacancy and collection losses have averaged 8% of Potential Gross Income (PGI) and are expected to continue at about the same rate in the future. Annual expenses totaling $117,000 include: Property taxes = $2,000 Property management = $7,000 Interest expense = $72,000 Swimming pool = $5,000 Professional fees = $8,000 Other expenses = $23,000 There is a monthly mortgage payment of $10,000 per month. Out of the $10,000 mortgage, $6,000 is interest expense and $4,000 is repayment of principal. Assuming a capitalization rate of 9%, what is the market value of the Chesapeake Bay complex? A) $1,941,422 B) $2,884,140 C) $3,560,667 D) $4,360,667

Rationale The correct answer is "D." Gross rental receipts ($650 x 60 x 12) = $468,000 plus non-rental income ($625 x 12) = $7,500 equals potential gross income (PGI) ($468,000 + $7,500) = $475,500. PGI minus vacancy and collection losses [$475,500 - (.08 x $475,500)] = $437,460 equals Effective Gross Income (EGI). EGI minus expenses equals net income $437,460 - $117,000 = $320,460. Next, determine net operating income by adding interest and depreciation expense back to net income. NOI = $320,460 + $72,000 interest + $0 depreciation = $392,460. Market value = $392,460 ÷ .09 = $4,360,667

Physical assets might be suitable as an investment in the portfolio of an investor looking for: A) Deflationary hedges. B) Stability of periodic cash flows. C) Short-term investments. D) Long-term capital gains.

Rationale The correct answer is "D." Hard assets are generally considered a hedge against inflation, which will lead to price appreciation and potential capital gains.

Jack Rich has an investment portfolio equally divided among the following funds: Energy sector fund, Bond Unit Investment Trust (25-year average maturity), and a Money Market fund. He is a buy-and-hold investor. Which of the following risks is his portfolio exposed? I. Business risk. II. Interest rate risk. III. Political risk IV. Purchasing power risk. A) I and III only. B) II and IV only. C) I, II and III only. D) I, III and IV only.

Rationale The correct answer is "D." Interest rate risk does not affect a bond investor if he or she holds the securities to maturity. This is how unit investment trusts are structured. The energy sector will be directly impacted by regulatory influences of a political nature.

According to fundamental analysis, which phrase best defines the intrinsic value of a share of common stock? A) The par value of the common stock. B) The book value of the common stock. C) The liquidating value of the firm on a per share basis. D) The discounted value of all future dividends

Rationale The correct answer is "D." Intrinsic value is the discounted value of a future stream of cash flows. In the case of a stock, its dividends.

Which portfolio manager has out performed the market, if the market had a standard deviation of 18.27%, a return of 10.23%, and the risk-free rate was 2.76% over the period in consideration? A) Manager A, over the period under consideration, had a portfolio which returned 12.12% with a standard deviation of 23.51% B) Manager B, over the period under consideration, had a portfolio which returned 13.3% with a beta of 1.44 C) Manager C, over the period under consideration, had a portfolio which returned 9.23% with a beta of .89 D) Manager D, over the period under consideration, had a portfolio which returned 15.12% with a standard deviation of 29.85%

Rationale The correct answer is "D." Keep in mind we want the Treynor or Sharpe indexes used here and the beta of the market is one. With this in mind, we must first calculate the value of the indexes for the market in both the Treynor measure and the Sharpe measure. This being done, we calculate each portfolio for either Treynor Index (if they use beta) or Sharpe Index (if they use standard deviation as a measure of risk). Only manager D beat the calculated market figures. Market Sharpe = 10.23 - 2.76 18.27 = 0.4088 Treynor = 10.23-2.76 1 = 7.47 A Sharpe = 12.12-2.76 23.51 = 0.3981 Treynor = No Beta Provided B Sharpe = No standard deviation provided Treynor = 13.3-2.76 1.44 = 7.3194 C Sharpe = No standard deviation provided Treynor = 9.23-2.76 0.89 = 7.2696 D Sharpe = 15.12-2.76 29.85 = 0.414 Treynor = No Beta Provided

Which of the following is not an appropriate match? A) Classification by time: Spot markets. B) Classification by type of claim: Equity markets. C) Classification by participants: Mortgage markets. D) Classification by products: Money markets.

Rationale The correct answer is "D." Money market securities are short-term instruments categorized by time considerations, not product. Look at this from the product to determine the classification. For example, money markets and spot markets are classified as according timing because they are either short term maturities or current price. The common component when classifying these type of securities is timing. Equity and debt markets can be classified as to the order of claims in the event of liquidation. "Type of claims" simply refers to debt vs. equity and which is more senior. Bond markets, which include mortgage bonds, are divided into short, intermediate and long term markets. Each market has participants that prefer different segments of the yield curve. A participant in this case is an insurance company, bank, manufacturing company, etc. Different participants will prefer mortgage bonds over shorter term maturities.

Which of these bonds initially immunizes a bond portfolio if the investors time horizon is 8 years? A) 20 year zero coupon B) Series of Tbills C) Coupon paying bond maturing in 8 years D) Coupon paying bond maturing in 10 years

Rationale The correct answer is "D." Option "A" - The term / duration is 20 years, which is well beyond the investors time horizon. Option "B" - The duration is too short (less than 12 month). Option "C" - The duration is something less than 8 which is too short. Option "D" - This is the only bond where the duration may equal the investor's time horizon, so it is the best answer.

Match the investment characteristic(s) listed below which describe(s) an open-end investment company. A) Only passive management of the portfolios. B) Shares of the fund are normally traded in major secondary markets. C) Both "A" and "B." D) Neither "A" nor "B."

Rationale The correct answer is "D." Option "A" is incorrect because open-end funds are both passively and actively managed. Option "B" is incorrect because open-end fund shares are traded directly with the fund, not on the secondary market.

Haley Mills has been a client of yours for quite some time. She recently unearthed some share certificates that her grandmother had left to her a few years ago. These shares that Grandma Mills bought were from an Internet start up. The timing was perfect, because the firm was about to undertake another stock offering and Haley had preemptive rights. Of the firm's initial 1,700,000 share offering, Grandma Mills had invested enough to buy 10,000 shares at $11 per share. The new offering was an 850,000 share offering at $87 per share. If Haley fully exercised her preemptive rights, how much total cash would she pay for the shares in this new offering? A) $55,000 B) $217,500 C) $385,750 D) $435,000

Rationale The correct answer is "D." The 10,000 share purchase of the 1,700,000 share initial offering was .59% (10,000 ÷ 1,700,000). That amount relative to this new offering of 850,000 shares was equivalent to 5,000 (850,0000 X .59) shares if all preemptive rights were exercised. This 5,000 shares times the $87 offering price means that to fully exercise this right, Haley would require $435,000.

Your client has asked you to assist her in examining possible additions to her bond portfolio. She has expressed a desire to minimize risk at this stage in her planning process, and to assure income beginning at the point of her retirement, and lasting throughout. She has a tentative retirement date in seven years at age 65. She will then have an eighteen year life expectancy. Which of the following is an appropriate addition to her current portfolio? I. 25-year AAA-rated corporate bonds with a seven-year maturity. II. 20 year AAA-rated municipal bonds with a seven-year duration. III. 25-year AAA-rated corporate zeroes with a seven-year duration. IV. 20-year US Treasury zeroes with a seven-year maturity. V. 25-year AAA-rated corporate bonds with a seven-year duration. A) I, III and V only. B) II, III and V only. C) III and IV only. D) V only.

Rationale The correct answer is "D." The client is looking for income to begin in 7 years. Therefore anything maturing in 7 years will not provide that income. Zeroes provide no income. She wants something out 25 years, not 20 years. Thus, option "V" is the only appropriate answer.

Put option sellers do best if the market price of the stock: A) Falls. B) Rises. C) Falls or remains at the same price. D) Rises or remains at the same price.

Rationale The correct answer is "D." The put option seller looks for the security price to rise and opposites the position of the put buyer who looks for a falling security price. If the price rises or stays the same, the put seller keeps the premium.

Use the following information to answer the question: Stock Beta ERR Amt Invested A 1.4 15% $10,000 B 1.2 12% $15,000 C 0.9 9% $11,000 Your client has expressed a desire to reduce the risk in her portfolio without reducing the expected rate of return. Your recommendation would be as follows: I. Increase the amount of stock A and decrease the amount of stock C. II. Increase the amount of stock B and decrease the amount of stock C. III. Increase the amount of stock C and decrease the amount of stock A. A) III only. B) I and II only. C) I and III only. D) None of the above.

Rationale The correct answer is "D." The risk in this portfolio cannot be reduced without a commensurate reduction in return based on the information given. To reduce risk, you need a lower weighted average beta, than the current portfolio allocation. To accomplish this, you must increase the amount invested in C (or maybe B too), but the point is you are increasing the amount invested in a security with a lower expected return thereby decreasing the expected return of the portfolio.

What is the geometric rate of return for a stock that has experienced the following prices over a four-year period? Year 1 = $20 Year 2 = $32 Year 3 = $24 Year 4 = $28 A) 17.23% B) 15.78% C) 13.82% D) 11.88%

Rationale The correct answer is "D." There are many ways to solve this, but here is the quickest: N=3 i=? PV=<20> PMT=0 FV=28 Assume you paid $20 for the stock today and three years later, it is trading at $28.

Your client is considering the two stocks described below. Assume for this question that the risk-free rate is 6%, the expected return on the market is 14%, and the market's standard deviation is 18%. Stock A price per share = $18 Stock A annual dividend = $2 Stock A dividend growth rate = 3% Stock A Beta = 1.1 Stock A standard deviation = 21% Stock A realized return over past 12 months = 15% Stock B price per share = $12 Stock B annual dividend = $1.50 Stock B dividend growth rate = 4% Stock B beta = 0.88 Stock B standard deviation = 14% Stock B realized return over past 12 months = 12.5% Which stock would you recommend your client purchase, and why? A) Stock A, because its intrinsic value is greater than Stock B's. B) Stock A, because its required return is greater than Stock B's. C) Stock A, because its risk-adjusted return is greater than Stock B's. D) Stock B, because it is selling for less than its intrinsic value.

Rationale The correct answer is "D." Though there is a great deal of information here, one should be looking at the intrinsic values of the stocks and comparing them to the market prices of the stocks for over valuation or under valuation. A market selling price above intrinsic value is overvalued. A market price below the intrinsic value means the stock is undervalued and may be considered for purchase. The first step in this problem is to compute the required rate of return using the CAPM formula: R = Rf + B * (Rm - Rf) Stock A =14.8% = 6 + 1.1 * (14-6) Stock B = 13.04% = 6 + .88 * (14-6) Now that we have the required rate of return on the securities based on their volatility, I would utilize the Gordon Growth Model (or dividend model) to determine the price that the security should be at to be "fairly valued." V = D * (1 + g) / (R - g) Stock A =17.45 = [2 * (1+.03)] / (0.148-.03) Stock B = 17.25 = [1.5 * (1+.04)] / (0.1304-.04) So the "fair value" for our investor is $17.45 for stock A and it is trading at $18. Therefore the stock is overvalued (trading higher than its value). Stock B is valued at $17.25 for our investor and is trading at $12. Therefore the stock is undervalued (trading lower than its value).

Which of the following have been repurchased by the corporation? A) Unissued shares. B) Repurchased shares. C) Authorized shares. D) Treasury shares.

Rationale The correct answer is "D." Unissued shares have never been held by investors to be repurchased. There is no such thing as "repurchased" shares. Authorized shares may be unissued or outstanding shares, but not necessarily Treasury shares (which are those the company has repurchased).

Anson Crowe has a 20% required rate of return. He is considering investing in XYZ, Inc., which pays an annual dividend of $.64 and is projected to increase its earnings and dividends by 17% annually. The current market price is $36.50. What is the expected rate of return from XYZ stock and is it undervalued or overvalued given Anson's requirements? A) 15.0%; overvalued. B) 17.5; undervalued. C) 18.2%; overvalued. D) 19.1%; overvalued.

Rationale The correct answer is "D." Use the expected rate of return and the intrinsic value formulas to calculate the correct answer to this problem. Expected rate of return: Er = (D1 ÷ P) + g Intrinsic Value: V = (D1) ÷ (r - g) D1 = .64 x 1.17 = .7488 Expected Return = (.7488 ÷ 36.50) + .17 = .1905 or 19.05% Intrinsic Value = .7488 ÷ (.20 - .17) = $24.96

Strong Form

Rejects - historical, public, and inside information Supports - none

Investment Policy Statement: The IPS establishes "RR TTLLU"

Risk, Return, Taxes, Timeline, Liquidity, Legal, and Unique circumstances - It does NOT include investment selection

Calculating gain or loss on a option position StOPS

St: stock gain or loss- if you own the underlying stock O: Options gain or loss (intrinsic value P: Premium paid (-) or received (+) S: Shares controlled or owned (IMPORTANT NOTE) When SELLING a put or call, the investor would have to buy the put or call option to close out the position. You must calculate this as a LOSS per share and a premium received.

What measure of risk does the CML use?

Standard deviation

Security Market Line (SML)

The SML intersects the y-axis at the risk-free rate of return The SML uses beta as its measure of risk - CML uses Standard deviation as its measure of riks - If a portfolio provides a return above the SML, it would be considered undervalued and should be purchased. -If a portfolio provides a return below the SML, it would be considered overvalued and should not be purchased

Best Efforts -Who carries the risk of the issue not selling?

The firm

Dividend Payout Ratio Dividend Payout Ratio = Common stock dividend/ EPS

Typically the higher the DPR, the more mature the company -A high DPR may also indicate the possibility of the dividend being reduced -A low DPR may indicate that the dividend may increase, there increasing the stock price

Treynor Index

a risk adjusted performance measure that uses Beta

Belief perseverance leads to:

avoiding changes to their belief in a investment, even though new information contradicts their original premise for investing -sticking to a flawed approach

Herd Mentality leads to:

buying high Selling low

Cognitive dissonance leads to:

minimizing or forgetting past losses - Exaggerating past gains

Semi-Strong Form

rejects - Historical and public information - Technical and fundamental analysis Supports - Inside information

Anchoring leads to:

returns or results that different than the investor expected - buying securities that have fallen in value because it must get back up to that recent high

Regret avoidance leads to:

selling winners too soon. Holding onto losers for too long

Firm commitment -Who carries the risk of issue not selling?

the underwriter

Sharpe Index

uses standard deviation


संबंधित स्टडी सेट्स

The Natural Gas Industry (Econ Exam 3)

View Set

Defining Networks With The OSI Model

View Set

𝘊𝘏𝘈𝘗𝘛𝘌𝘙 9 ~𝘉𝘜𝘚𝘐𝘕𝘌𝘚𝘚

View Set

Science SOL Most Missed Questions

View Set

Chapter 16&17 international marketing

View Set